Calendrier de l’Avent

1234689

Réponses

  • Namiswan
    Modifié (December 2022)
    Oups, j'ai failli oublier...

    Jour 13

    Résoudre dans les entiers naturels l'équation
    $$x^y=y^x+7.$$
    En bonus, vous pouvez résoudre aussi 
    $$2x^y=y^x+7$$
    (mais c'est presque la même chose).
  • Une etude de fonction montre que $x^y>y^x$ (en nombres entiers) ssi $x=2$ et $y>4$ ou $x>y\geqslant3$. 


    Pour $x=2$, on a $x^5-5^2=7$ et une etude de fonction montre que $2^y-y^2>7$ pour $y>5$.


    Essayons a present $x\geqslant3$ ; une etude de fonction montre que $y\geqslant x+1\in\N\mapsto x^y-y^x$ est strictement croissante ; si l'on montre que $x^{x+1}-(x+1)^x>7$, on n'aura plus de nouvelle solution.


    Or, pour $x\geqslant5$ entier, $x^{x+1}-(x+1)^x=x^x\times\big(x-(1+\frac1x)^x\big)\geqslant x^x(x-{\rm e})\geqslant2x^x>7$. Il reste a essayer $x=3$ (mais $3^4-4^3>7$) et $x=4$ (mais $4^5-5^4>7$).

  • @john_john il reste la solution "triviale" :mrgreen:  :

    $$x=8, y=1$$
  • john_john
    Modifié (December 2022)
    Exact ! J'étais parti de $x,y>1$;

    À propos de solution triviale, Bombieri dit un jour, en présence de Roger Apery, que ${x\choose n}+{y\choose n}={z\choose n}$ n'a pas de solution non triviale si $n<4$ ; le lendemain matin, Apery montre à Bombieri la solution $n = 3, x = 10, y = 16, z = 17$. Bombieri, vexé : j'avais dit non triviale.
  • Namiswan
    Modifié (December 2022)
    Ca me va, même si certains points mériteraient des détails techniques

    (en particulier la croissance de $y\mapsto x^y-y^x$ pour $y\geq x+1$).

  • john_john
    Modifié (December 2022)
    Merci, AD, pour les accents ! Demain ou vendredi, je retrouverai mon brave clavier français :)
    [À ton service. :) AD]

    Namiswan : pour étudier le signe de la dérivée $A-B$, où $A=x^y\ln x$ et $B=xy^{x-1}$, j’étudie le signe de $\ln A-\ln B$ en tant que fonction auxiliaire.
  • Plus precisement, $x^y\ln x-xy^{x-1}$ est du meme signe que $f(y)=y\ln x+\ln\ln x-\ln x-(x-1)\ln y$ ; or $f'(y)=\ln x-(x-1)/y\geqslant0$ car car $(x-1)/y\leqslant1$.

     

    Donc, $f$ est croissante et $f(y)\geqslant f(x)=\ln\ln x\geqslant0$.
  • Jour 14

    Soit $(a_n)\in\Bbb C^{\mathbb{N}^*}$ une suite sommable telle que, pour tout $m\in \mathbb{N}^*$, $\displaystyle \sum_{k=1}^\infty a_{mk} =0$. Montrer que : $\forall n\in \mathbb{N}^*,\; a_{n} =0$.
  • Bibix
    Modifié (December 2022)

    Soit $n \in \mathbb{N}^*$, on note $\mathcal{P}$ l'ensemble des nombres premiers et on pose pour $p \in \mathcal{P}$ $A_{p} = \{m \in \mathbb{N}^* \mid p n \mid m, \, \forall q \in \mathcal{P}, q < p \Longrightarrow q n \not\mid m\}$. On note $(p_i)_{i \in \mathbb{N}^*}$ la suite croissante de tous les nombres premiers. On a pour tout $(p,q)\in\mathcal{P}^2, q \neq p \Longrightarrow A_p \cap A_q = \emptyset$ et $$A_p = \{n p k \mid k \in \mathbb{N}^*\} \setminus \bigcup_{q < p, q \in \mathcal{P}} A_q.$$

    Donc on obtient pour tout $l, r\in \mathbb{N}$ et $i \in (\mathbb{N}^*)^r$ (avec la sommabilité)

    $$\sum_{m \in A_{p_{l+1}}, p_{i_1} \dots p_{i_r} \mid m} a_m = \sum_{k = 1}^{\infty} a_{n p_{l+1}p_{i_1} \dots p_{i_r} k} - \sum_{j = 1}^l \sum_{m \in A_{p_j}, p_{l+1}p_{i_1} \dots p_{i_r} \mid m} a_m = -\sum_{j = 1}^l \sum_{m \in A_{p_j}, p_{l+1}p_{i_1} \dots p_{i_r} \mid m} a_m.$$

    Par récurrence forte sur $l$, on obtient pour tout $r \in \mathbb{N}, i \in (\mathbb{N}^*)^r$, $\sum_{m \in A_{p_l}, p_{i_1} \dots p_{i_r} \mid m} a_m = 0$ et cela pour tout $l \in \mathbb{N}^*$. En particulier, on obtient avec $r = 0$ la formule

    $$\sum_{m \in A_{p_l}} a_m = 0.$$

    De plus, on a $\bigcup_{p \in \mathcal{P}} A_p = \{m \in \mathbb{N} \mid m \geqslant n + 1, n \mid m\}$. On en déduit par sommabilité que

    $$\sum_{m \geqslant n + 1, n \mid m} a_m = \sum_{p \in\mathcal{P}} \sum_{m \in A_p} a_m = \sum_{p \in\mathcal{P}} 0 = 0.$$

    Donc $\sum_{k = 2}^{\infty} a_{n k} = 0$. Et donc d'après l'hypothèse, $a_n = 0$.

  • Namiswan
    Modifié (December 2022)

    Soit $E_m=\{mk, k\in\N^*\}$. L'hypothése dit que pour tout entier $m$, 

    $$\sum_{n=1}^{+\infty} \chi_{E_m}(n) a_n=0.$$

    Pour pour $K\in\N^*$ posons $F_K$ l'ensemble des entiers divisibles par aucun des $K$ premiers nombres premiers.

    Si on note $(p_k)_{k\geq 1}$ la suite des nombres premiers, on a $F_K=E_{p_1}^c\cap \cdots \cup E_{p_K}^c$. Donc $\displaystyle \chi_{F_K}=\prod_{k=1}^K(1-\chi_{E_{p_k}})$. En développant et en utilisant que toute intersection d'ensembles $E_{p_i}$ est un ensemble $E_m$ (par Gauss) on déduit que $\chi_{F_K}$ est combinaison linéaire des ensembles $E_m$.

    Donc pour tout $K$, $$\sum_{n=1}^{+\infty} \chi_{F_K}(n) a_n=0.$$ 

    En faisant tendre $K$ vers $+\infty$, on obtient par convergence dominée que $a_1=0$.

    Finalement, pour tout entier $k$, la suite définie par $b_n=a_{kn}$ vérifie la même hypothèse, donc $b_1=0$, donc $a_k=0$.

  • Calli
    Modifié (December 2022)
    @Namiswan : Joli !

    @Bibix : Je ne comprends pas l'égalité suivante dans ta réponse.
    $$ \sum_{m \in A_p} a_m = \sum_{p \in\mathcal{P}} \left(\sum_{k = 1}^{\infty} a_{k p} - \sum_{q \in \mathcal{P}, q < p} a_{k p q}\right)$$ J'aurais plutôt dit $ \sum\limits_{p \in\mathcal{P}} \Big(\sum\limits_{k = 1}^{\infty} a_{knp} - \sum\limits_{q \in \mathcal{P}, q < p} a_{kn p q}\Big)$ avec des $n$ en plus dans les indices. Et il me semble que certains indices $knpq$ peuvent être comptés plusieurs fois, pour plusieurs $q$.
  • Bibix
    Modifié (December 2022)
    Désolé pour la grossière erreur dans mon égalité. J'ai corrigé, ça devrait être bon maintenant.
  • Je crois qu'il y a toujours un souci @Bibix.
    $A_p = \{n p k \mid k \in \mathbb{N}^*\} \setminus \bigcup_{q < p, q \in \mathcal{P}} A_q.$ n'est pas équivalent à $\{n p k \mid k \in \mathbb{N}^*\} = \bigsqcup_{q \leqslant p, q \in \mathcal{P}} A_q$ (avec union disjointe). Et en l'occurrence la deuxième propriété est fausse, donc on ne peut pas écrire $$ \sum_{m \in A_{p_{i+1}}} a_m = \sum_{k = 1}^{\infty} a_{n p_{i+1}k} - \sum_{j = 1}^i \sum_{m \in A_{p_j}} a_m .$$
  • raoul.S
    Modifié (December 2022)
    Voici ma preuve snob (et monotone :mrgreen: ) pour le jour 14

    On considère $\N^{*}$ munit de la tribu $\mathcal{P}(\N^{*})$ et de la mesure du cardinal. Notons $f:\N^{*}\to \C, n\mapsto a_n$, cette fonction est intégrable car $(a_n)$ est sommable. Soit de plus pour $m\in \N^{*}$, $I_m:=m\N^{*}$ (les multiples de $m$).
    1) On remarque facilement que les $I_m$ engendrent la tribu $\mathcal{P}(\N^{*})$. En effet pour $m\in \N^{*}$ $\{m\}=I_m\setminus (\bigcup_{k=2}^{\infty} I_{km})$, ce qui prouve que les singletons sont dans la tribu engendrée et de là on obtient $\mathcal{P}(\N^{*})$.

    2) L'ensemble $\mathcal{M}:=\{A\subset \N^{*} \mid \displaystyle \int_A f=0 \}$ est une classe monotone. (C'est quasiment immédiat, pour vérifier l'axiome 3 des classes monotones utiliser la convergence dominée). 

    3) L'ensemble $\{I_m\mid m\in \N^{*}\}$ est stable par intersections finies (évident car $I_m\cap I_n=I_{ppcm(m,n)}$.

    4) L'ensemble $\{I_m\mid m\in \N^{*}\}$ est contenu dans $\mathcal{M}$ par hypothèse. On peut donc appliquer le lemme de la classe monotone qui nous assure que la tribu engendrée par les $I_m$ (qui est $\mathcal{P}(\N^{*})$) est contenue dans $\mathcal{M}$. Par conséquent, $\mathcal{M}=\mathcal{P}(\N^{*})$ et en particulier pour tout entier $m\in \N^{*}$, $a_m=\int_{\{m\}} f=0$.
  • J'aime bien tes preuves snobs @raoul.S. Toujours amusantes.  :)
  • @Calli : tu me lacherais le jour 16 stp ?
  • Merci @Calli content qu'elles te plaisent  :)
  • D'accord @Magnéthorax. Mais attention, c'est le dernier jour que je laisse ! (normal, j'ai déjà donné tous les autres :mrgreen:
  • AD
    AD
    Modifié (December 2022)
    John-John m'a chargé de publier son J15. AD
    ----------------------------------------------------------------------------------------------------
    john_john  John-John

    Jour 15

    On désigne par $I$ le segment $[0,1]$, par $E$ l'espace vectoriel $C(I,\C)$ et par $F$ le sous-espace formé des solutions maximales de l'EDO $$y''+p(x)y'+q(x)y=0,$$ $p$ et $q$ sont éléments de $E$. On désigne par $\mathcal C$ l'endomorphisme $f\in E\mapsto f\circ\varphi\in E$, où $\varphi$ est une bijection continue de $I$  sur $I$, et l'on suppose que $\mathcal C$ laisse stable $F$.
    Étudier l'endomorphisme $\Gamma$ induit par $\mathcal C$ sur $F$.

    Comme l'on dit en Lorraine (mais pas seulement ici), je ne vous dis pas quoi ; vous obtiendrez sans doute d'autre propriétés que celles que j'ai trouvées.
    Remarque : on peut généraliser avec les solutions d'une EDO linéaire d'ordre quelconque, résolue en $y^{(n)}$ ; cela ne fait que compliquer les notations.
  • L'exo du J15 ne bouge pas beaucoup (pour l'instant) ; peut-être est-il un peu rude, mais il faut dire que j'ai trouvé dans la base de donnée fournie dans Approach0.xyz tous autres les exos que je pensais inédits  >:)
    Alors, je vous propose de trouver ci-dessous quelques propriétés à démontrer. En deux temps.

    Si $\varphi$ croît, établir successivement que $1$ est la seule valeur propre (complexe) de $\Gamma$, puis que $\Gamma$ est l'identité, puis que $\varphi$ est l'identité.

  • Deuxième temps.

    Si $\varphi$ décroît, montrer que $\Gamma$ est une involution mais sans être une homothétie.

  • marco
    Modifié (December 2022)
    Pour le cas où $\phi$ croît, on a $\phi(0)=0$. On raisonne au point $x=0$. Soit $z=y \circ \phi$, on a $z''(x)+p(x)z'(x)+q(x)z(x)=0$. $z'=\phi' \cdot y' \circ \phi$, et $z''=(\phi')^2 (y'' \circ \phi)+\phi'' \cdot y' \circ \phi)$. En $x=0$, on a $(\phi'(0))^2y''(0)+\phi''(0)y'(0)+p(0)\phi'(0)y'(0)+q(0)y(0)=0$ qui est l'équation $(1)$.
    Ceci est valable pour tout $y(0),y'(0) \in \R$, car d'après Cauchy-Lipschitz, il y a une solution sur $I$ pour toutes conditions initiales (il me semble).
    De plus, on a $y''(0)+p(0)y'(0)+q(0)y(0)=0$. On remplace la valeur de $y''(0)$ dans $(1)$.
    Donc $(-\phi'(0)^2p(0)+\phi''(0)+p(0)\phi'(0))y'(0)+(q(0)-\phi'(0)^2q(0))y(0)=0$, pour tout $y(0), y'(0)$.
    Donc $q(0)-\phi'(0)^2q(0)=0$ et $-\phi'(0)^2p(0)+\phi''(0)+p(0)\phi'(0)=0$.
    Si $q(0) \neq 0$, on a $\phi'(0)=1$, car $\phi$ croît.
    Donc $\phi''(0)=0$.
    Donc, pour tout $y$ solution de l'équation différentielle, $z=y \circ \phi$ vérifie $z'(0)=\phi'(0)y'(0)=y'(0)$ et de même, $z''(0)=\phi'(0)^2y''(0)+ \phi''(0)y'(0)=y''(0)$. Les conditions initiales étant les mêmes $z=y$.
    Donc l'endomorphisme $y \mapsto y \circ \phi$ est l'identité.
    Si $q(0)=0$ je ne sais pas.
  • Bonjour, Marco,
    a) Montrer que $y\circ\varphi=y$ par unicité est une jolie idée !
    b) Je n'ai pas dit que $\varphi$ est de classe $C^2$, mais peut-être l'est-il automatiquement :)
    c) Je ne sais pas si l'on peut rattraper ce cas où $q(0)=0$ (densité ???) ; en tout cas, j'ai procédé de façon  très différente (60% d'Analyse et 60% d'Algèbre, mais, comme le disait César, cela dépend de la taille des pour cent)
  • marco
    Modifié (December 2022)
    Pour montrer que $\phi$ est de classe $C^2$.
    Si $y \in F$, $z=y \circ \phi \in F$, donc $z$ est dérivable deux fois. Donc $\lim_{x \to x_0} \frac{z(x)-z(x_0)}{x-x_0}$ existe pour tout $x$ de $I$. Or $\frac{z(x)-z(x_0)}{x-x_0}=\frac{y\circ \phi(x)-y \circ \phi(x_0)}{\phi(x)-\phi(x_0)} \times \frac{\phi(x)-\phi(x_0)}{x-x_0}$. Donc, comme $y$ est dérivable en $\phi(x_0)$, si $y'( \phi(x_0))\neq 0$, alors $\frac{\phi(x)-\phi(x_0)}{x-x_0}$ admet une limite en $x_0$, donc $\phi$ est dérivable en $x_0$ (on peut supposer $y'(\phi(x_0))\neq 0$, car on peut choisir les conditions initiales en $\phi(x_0)$). Donc $\phi'(x_0)=\frac{z'(x_0)}{y'(\phi(x_0)}$. Donc $\phi'$ est continu car $z'$, $y'$ et $\phi$ le sont. De plus, $z'$, $y'$ et $\phi$ sont de classe $C^1$, donc $\phi'$ est $C^1$. Donc $\phi$ est $C^2$.
  • Exactement ! Pour tout $x_0$, il existe une des solutions dont la dérivée est non nulle en ce point ; donc, la partie réelle (ou la partie imaginaire) d'une telle solution est localement un difféomorphisme de classe C$^2$.
  • gebrane
    Modifié (December 2022)
    John le prix à payer lorsque, on donne une question inaccessible et non référencée par le moteur diabolique, c'est qu'on soit obliger de rédiger une solution détaillée par soi-même. On attend ta rédaction  >:) à 60% algèbre et 60% analyse (même si ça dépasse les 120%)  
    Le 😄 Farceur


  • john_john
    Modifié (December 2022)
    Bonsoir, gebrane,
    c'est ce que j'ai fait dans la journée, quoique marco ait déjà répondu à 80%.

    a) Commencons par le cas où $\varphi$ est (strictement) croissante.

     

    Soit alors $\lambda$ une valeur propre complexe de $\Gamma$ et $f\in F$ un vecteur propre associé (on a donc $|\lambda|=1$ car $f$ et $f\circ\varphi$ ont même image) ; on choisit un $a\in I$ tel que $f(a)\neq0$ et l'on définit la suite récurrente $x_0=a,x_1=\varphi(x_0),\dots$ On sait que la suite $(x_n)$ est monotone et admet donc une limite $\ell$. Puisque $f\circ\varphi=\lambda f$, on a $f(x_n)=\lambda^nf(a)$ et donc $\lambda^n$ tend vers $f(\ell)/f(a)$ : cela montre que $\lambda=1$ car $\sum(\lambda^{n+1}-\lambda^n)$ converge.

     

    Il s'ensuit que $\Gamma={\rm Id}_F$ : avec les choix précédents, si $(f,\,g)$ est une base de $F$, on a $\Gamma(g)=g+\alpha f$ pour un certain $\alpha$ et donc $\Gamma^n(g)=g+n\alpha f$ : donc $g(x_n)=g(a)+n\alpha f(a)$ et donc $\alpha=0$ puisque la suite $(g(x_n))$ est bornée.

     

    Maintenant, on a même $\varphi={\rm Id}_I$ : sinon, on choisit un $a\in I$ tel que $\varphi(a)\neq a$ et, pour tout $h\in F$, on a $h(x_n)=h(x)$ de sorte que $\ell$ est limite d'une suite injective en laquelle $h$ prend les mêmes valeurs. Ainsi, $h'(\ell)=0$ et c'est absurde car le wronskien d'une base de $F$ s'annule en ce point.

     

    b) Si $\varphi$ est strictement décroissante, alors on applique le  a) à $\varphi\circ\varphi$ : l'on voit que $\varphi$ est une involution de $I$ et $\Gamma$ une involution de $F$.

     

    Bizarrement, $\Gamma$ ne peut être $\pm{\rm Id}$ : si $\Gamma=-{\rm Id}$, on choisit $c\in I$ tel que $\varphi(c)=c$ et l'on a donc $h(c)=-h(c)$ pour tout $h\in F$ (contradiction avec le wronskien).

     

    Si $\Gamma=+{\rm Id}$, avec un point fixe $c$, on a cette fois $h'(c)\varphi'(c)=h'(c)$ et donc $h'(c)=0$ car $\varphi'(c)<0$ (contradiction avec le wronskien).

     

    Comment cela ? On n'a pas dit que $\varphi$ est dérivable ??? Mais si : pour tout $x\in I$, on choisit $f\in F$ telle que $f'(x)\neq0$ (avec par exemple partie réelle de $f'(x)\neq0$) et l'on pose $g=f\circ\varphi$. On a aussi $f_1\circ\varphi=g_1$ si $f_1$ et $g_1$ sont les parties réelles respectives de $f$ et de $g$. Localement, $f_1$ est un difféomorphisme de classe~C$^1$ d'un voisinage de $x$ sur son image et l'on a localement $\varphi=(f_1)^{<-1>}\circ g_1$.

  • marco
    Modifié (December 2022)
    Merci pour la solution ! Je n'avais pas réalisé que $\Gamma$ avait un vecteur propre, car c'est un endomorphisme.
  • De rien, marco

    Nota bene : l'idée de la suite récurrente n'est pas de moi ; j'avais une méthode plus tordue.
  • Peut-on savoir la source de cette question ?
    Le 😄 Farceur


  • i.zitoussi
    Modifié (December 2022)
    @john_john Pas sûr d'avoir compris tous les arguments, mais, pire, pas sûr non plus de pouvoir résumer le résultat. Donc, si j'ai bien compris...
    On cherche le groupe, disons $G$, des bijections $\varphi:[0,1]\to [0,1]$ qui préservent l'ensemble $F$ des solutions de l'équation (dans le sens où $f$ solution $\Rightarrow$ $f\circ\varphi$ solution).
    (a) La seule bijection croissante possible est l'identité. (b) Si $\varphi_1$ et $\varphi_2$ sont décroissantes, alors par (a) $\varphi_1\circ\varphi_2=\mathrm{id}$ puisque croissante. J'ai l'impression que cela implique que $G$ a au plus deux éléments, $\mathrm{id}$ et une éventuelle involution décroissante $\varphi$. Après tu as montré que si une telle $\varphi$ existe, l'endomorphisme induit sur $F$ prend les deux valeurs propres $+1$ et $-1$, donc il existe une base (une j_j-basis) $\{f_1,f_2\}$ de $F$ telle $f_1\circ\varphi = +f_1$ et $f_2\circ\varphi = -f_2$. Une question que je me pose est si ce $\varphi$ d'ordre 2 existe toujours. Ou se peut-il au contraire que $G=\{\mathrm{id}\}$ ? J'espère bien entendu ne pas avoir dit de c...
    Après je bloque.
  • Magnéthorax
    Modifié (December 2022)

    Jour 16

    (J'aime à penser que c'est inédit parce que je l'ai trouvé dans mon coin, mais toute contradiction m'intéresse.)

    On note $\mathcal{C}$ l'algèbre des fonctions continues $f:\left[0,1\right]\to\mathbf{R}$. Soit $T:\mathcal{C}\to\mathcal{C}$ l'application définie par $$T\left(f\right)\left(x\right)=f\left(x\right)\left(2-\left(1+x^2\right)f\left(x\right)\right).$$
    On note $\left(f_n\right)$ la suite de $\mathcal{C}$ définie par $$f_n =\begin{cases} T\left(f_{n-1}\right) & \text{si } n>0 \\1/2 & \text{si } n=0\end{cases}$$
    et on note $\left(I_n\right)$ la suite réelle définie par $I_n=\int_0^1 f_n\left(x\right)\, dx $.
    Montrer que $\left(I_n\right)$ est une suite de rationnels qui converge en croissant, déterminer sa limite $\ell$ et, pour tout $n\in\mathbf{N}$, majorer $\ell-I_n$ par le terme général d'une "suite simple qui converge rapidement vers $0$". Expliciter $I_n$.
  • Bibix
    Modifié (December 2022)

    On montre facilement par récurrence que pour tout $n \in \mathbb{N}$, on a : 

    $$\forall x \in [0,1], \quad 0 \leqslant f_n(x) \leqslant \frac{1}{1 + x^2}.$$

    On en déduit que $f_{n}(x) = f_{n-1}(x) (2 - (1+x^2)f_{n-1}(x)) \geqslant f_{n-1}(x)$. Ainsi, $(I_n)$ est une suite croissante. De plus, pour $x$ fixé, $g_x : u \longmapsto u(2 - (1+x^2)u)$ admet comme points fixes $0$ et $\frac{1}{1+x^2}$. Comme la suite $(f_n(x))$ est croissante, strictement positive et majorée, on a

    $$\forall x \in [0,1], \quad \lim_{n \to +\infty} f_n(x) = \frac{1}{1 + x^2}.$$

    On obtient avec le théorème de convergence monotone que $\ell = \lim_{n \to +\infty} I_n = \int_0^1 \frac{1}{1 + x^2} dx = \frac{\pi}{4}$.

    Je vois comment trouver une estimation de la convergence, mais expliciter $I_n$ me parait compliqué... . Peut-être $I_n = \frac{1}{2} \prod_{1 \leq k \leq n} \frac{2^{k+1}}{4k + 3}$...

  • bd2017
    Modifié (December 2022)
    Bonjour
    J'ai fait le même  que @Bibix avec peut être ceci en plus: $I_n=\dfrac{\pi}{4} - \dfrac{1}{2 ^{2^ n}} \int_0^1  \dfrac{(1-x^2)^{2^n}}{1+x^2}  dx$  mais je ne sais pas si c'est la réponse attendue pour la question "expliciter $I_n$"
     
  • john_john
    Modifié (December 2022)
    (Réponses à propos de J15)

    i.zitoussi : effectivement, ton résumé est exact ; je vais y réfléchir, mais je pense que $G$ peut très bien être réduit à l'identité : il n'y a aucune raison qu'une involution décroissante existe. Je n'avais pas remarqué que la bijection décroissante devait être unique.

    gebrane : source = bibi ; j'avais constaté que, lorsque l'intervalle est symétrique par rapport à $0$ (ce qui n'est pas le cas ici), on pouvait avoir des solutions paires et des solutions impaires, mais jamais une base de solutions paires ou de solutions impaires (le wronskien s'annulait alors en $0$). J'ai donc cherché à généraliser. Comme tu le vois, le sujet est riche, et j'en avais fait un sujet de concours ; l'idée d'utiliser des suites récurrentes vient de l'IG Edmond Ramis, dont je salue la mémoire.
    Tout ça parce que d'autres idées de sujet se trouvaient pistées par le moteur satanique  >:)>:)>:)
  • gebrane
    Modifié (December 2022)
    John dit il n'y a aucune raison qu'une involution décroissante existe. Prenons le cas où  p  et q  sont les fonctions nulles. Alors F est le s.e.v des applications affines,  on peut prendre dans ce cas $\phi(x)=x $ ou  $\phi(x)=1-x $ ils stabilisent F. Bien sûr  $\phi(x)=1-x $ est une involution décroissante 
    Le 😄 Farceur


  • @john_john Merci pour l'exercice, très intéressant. Ceci dit, si ça fait effectivement un beau sujet de concours, je trouve que ça mériterait d’être plus connu, voire même de faire partie du cours, en particulier en physique. J'ai fait Physique et n'ai jamais entendu parler de ça.
    Après je bloque.
  • Magnéthorax
    Modifié (December 2022)
    @Bibix et @bd2017 : vous avez trouvé la limite de $\left(I_n\right)$ :)

    Le reste est encore à faire, notamment pour la vitesse de convergence et l'explicitation des termes. Est-ce que je mets les réponses sans preuve ? Avec preuve ?


  • (J15) gebrane : certes, mais une involution décroissante peut ne pas exister. Si l'on prend $I=[0,\pi/2]$ et comme solutions les fonctions $x\mapsto x$ et $x\mapsto\cos x$ (ce sont bien les solutions d'une EDO linéaire homogène d'ordre $2$ résolue en $y''$), il n'existe pas de $\varphi$, involution décroissante. En effet, sinon, il existe des scalaires $a,b$ tels que, pour tout $x$, on ait $\varphi(x)=ax+b\cos x$. Pour $x=0$, on a $\varphi(0)=\pi/2$ et donc $b=\pi/2$. Ensuite, pour $x=\pi/2$, on obtient $a=0$. Donc, $\varphi(x)=\pi/2\cos x$ mais ce n'est pas une involution.
  • bd2017
    Modifié (December 2022)
    Bonsoir @Magnéthorax  je n'ai pas vérifié l'expression de $I_n$  que j'ai donnée. Mais je pense qu'elle est bonne et elle donne la vitesse de de CV. L'erreur est plus petite que $\dfrac{1 }{2 ^{2^n}}.$  La vitesse de CV est quadratique.
    Edit. Je dois poser mon exercice demain 17. Mais comme il y a coupure de courant, j'ai l'intention de le poser ce soir. Est ce OK?
     
  • Merci John.
    Le 😄 Farceur


  • bd2017
    Modifié (December 2022)

    Jour 17

    Montrer que pour tout $a,b,c \in \R$ on a $$ \sum_{cycl} (a^4 - a^3 b + 8 a^2 b^2 - 7 a^2 b c - a b^3 ) \geq 0.$$
     
  • Julia Paule
    Modifié (December 2022)
     J'ai le même résultat que @bd2017 pour le J16.
    $I_n=\dfrac{\pi}{4} - \dfrac{1}{2 ^{2^ n}} \int_0^1  \dfrac{(1-x^2)^{2^n}}{1+x^2}  dx$ et aussi $I_n = \dfrac{1}{2} \sum_{k=0}^{2^n-1} \dfrac{1}{2^k}  \int_0^1  (1-x^2)^k  dx$.

    Par contre, j'ai $I_1=\dfrac{2}{3}$, que je ne retrouve pas avec le calcul de @Bibix . J'ai $I_n= \dfrac{1}{2} \sum_{p=0}^{2^n-1}\dfrac {2^p}{(2p+1)\binom{2p}{p}}$.
  • Magnéthorax
    Modifié (December 2022)
    @bd2017 : on peut faire plus explicite, du moins sans symbole d'intégration.
    Si quelq'un connait cette méthode simple et "efficace" d'approximation rationnelle de $\pi$ par itération, je suis intéressé par une référence.

    Réponses pour le jour 16.
    Pour tout $n\in\N$, $f_n$ est polynomiale à coefficients rationnels et les bornes de l'intégrale définissant $I_n$ sont rationnelles. On note $g$ l'élément de $\mathcal{C}$ défini par $g\left(t\right)=1/\left(1+t^2\right)$. On a $g-T\left(f\right)=\left(g-f\right)^2/g$ dont on tire $$\forall n\in\mathbf{N} \qquad g-T^n \left(f\right)=g\left(1-\frac{f}{g}\right)^{2^n}$$ puis (somme géométrique) $$\forall n\in\mathbf{N}  \qquad T^n \left(f\right)=f\sum_{k=0}^{2^n-1} \left(1-\frac{f}{g}\right)^k$$ Donc $$\forall n\in\mathbf{N}, \forall t\in\left[0,1\right] \qquad f_n\left(t\right)=\frac{1}{2}\sum_{k=0}^{2^n-1} \left(1-\frac{1+t^2}{2}\right)^k$$ On obtient $$\forall n\in\mathbf{N}  \qquad I_n=\frac{1}{2} \sum_{k=0}^{2^n} \frac{1}{2^k} \int_0^1 \left(1-t^2\right)^k \,dt$$ puis (intégrales de Wallis) $$\forall n\in\mathbf{N} \qquad I_n=\frac{1}{2} \sum_{k=0}^{2^n} \frac{2^k}{\left(2k+1\right)\binom{2k}{k}}$$ On montre enfin $$\forall n\in\mathbf{N},\forall t\in\left[0,1\right] \qquad 0 \leq  g\left(t\right)-f_n\left(t\right) \leq \frac{1}{2^{2^n}}\left(1-t^2\right)^{2^n}$$ dont on déduit $$\forall n\in\mathbf{N} \qquad 0 \leq \frac{\pi}{4} -I_n \leq  \frac{1}{2^{2^n}}$$
  • Bibix
    Modifié (December 2022)

    Jour 18

    Soient $u = (u_n)_{n \in \mathbb{N}}$ une suite réelle bornée et $(k_n)_{n \in \mathbb{N}}$ une suite d'entiers naturels tel que $\displaystyle \sup_{i \in \mathbb{N}} |u_{i + k_n} - u_i| \to 0$ quand $n \to +\infty$.
    1) Montrer que si $k_n = P(n)$ avec $P \in \mathbb{Z}[X] \setminus \{0\}$ à coefficients positifs, alors $u$ est périodique et déterminer dans ce cas une période de $u$ en fonction de $P$.
    2) Si $k_n$ est quelconque, $u$ est-elle forcément périodique ?
    Comme ça concerne les définitions de presque périodicité non uniformes (celles qui ne servent quasiment à rien), peut-être que cet exercice est vraiment inédit. En tout cas, je n'ai pas réussi à le trouver avec approach0xyz. Et oui, ça se généralise avec des notions plus avancées, mais je n'ai mis que l'essentiel.
  • Magnéthorax
    Modifié (December 2022)
    @Julia Paule : j'ai la même expression que toi pour $I_n$; j'ignore si nous y parvenons par le même chemin.
    @bd2017 : pourrais-tu préciser la signification de la notation que tu emploies, stp ?
  • Je comprends que si f(a,b,c) est le polynôme en question la somme est f(a,b,c)+f(b,c,a)+f(c,a,b)....
  • bd2017
    Modifié (December 2022)
    Bonjour
    Concernant la notation que j'emploie, elle ne m'est pas personnelle car je l'ai rencontrée à plusieurs reprises. Pour simplifier on écrit seulement le premier terme, les autres s'obtiennent en remplaçant  a par b , b par c .....      
    Exemple
    $\sum_{cycl}  (a+  2 b - c)=  (a+  2b - c)  +(b+  2c  -a)  +  (c + 2a  -b)$

    Edit:  pas vu le message de @Boécien  mais c'est cela.   
     
Connectez-vous ou Inscrivez-vous pour répondre.